Mathcenter Forum

Mathcenter Forum (https://www.mathcenter.net/forum/index.php)
-   ปัญหาคณิตศาสตร์ ม. ต้น (https://www.mathcenter.net/forum/forumdisplay.php?f=31)
-   -   แนะนำการจัดรูป อสมการข้อนี้ด้วยครับ (https://www.mathcenter.net/forum/showthread.php?t=15221)

neo-freeman 19 ธันวาคม 2011 09:55

แนะนำการจัดรูป อสมการข้อนี้ด้วยครับ
 
ถ้า $ \frac{19}{20} < \frac{1}{a} + \frac{1}{b} + \frac{1}{c} < 1 $ แล้ว a+b+c มีค่าน้อยที่สุดเมื่อใด

...ลองจัดหลายแบบแล้ว แต่ยังไม่เห็นความสัมพันธ์ ของ a+b+c ที่จะมีค่าน้อยที่สุด ครับ :(

banker 19 ธันวาคม 2011 10:17

ยังเช้าอยู่ มาลองมั่วแก้หนาวดูครับ


$ \frac{19}{20} < \frac{1}{a} + \frac{1}{b} + \frac{1}{c} < 1 $


$ \frac{19 \times 6}{20 \times 6} < \frac{115}{120} < \frac{120}{120} $

$ \frac{19 \times 6}{20 \times 6} < \frac{1}{8} + \frac{1}{3} + \frac{1}{2} < \frac{120}{120} $

$a+b+c = 8+3+2 = 13$

neo-freeman 19 ธันวาคม 2011 10:50

ความสัมพันธ์ของตัวเลข ข้างบน หาได้อย่างไรครับ
- ลองสุ่ม
- สังเกตจากประสบการณ์ การทำโจทย์
- หรือมีหลักอย่างไร

ขอบคุณครับ

gon 19 ธันวาคม 2011 13:57

อ้างอิง:

ข้อความเดิมเขียนโดยคุณ neo-freeman (ข้อความที่ 128933)
ถ้า $ \frac{19}{20} < \frac{1}{a} + \frac{1}{b} + \frac{1}{c} < 1 $ แล้ว a+b+c มีค่าน้อยที่สุดเมื่อใด

...ลองจัดหลายแบบแล้ว แต่ยังไม่เห็นความสัมพันธ์ ของ a+b+c ที่จะมีค่าน้อยที่สุด ครับ :(

คุ้น ๆ นะครับ เหมือนจะเคยเห็นไม่นานมานี้ :huh: แต่ยังไม่ได้ลองทำ

อันนี้ผมสมมติว่า a, b, c เป็นจำนวนเต็มบวกเท่านั้นนะครับ ไม่ได้เป็นจำนวนจริงหรือจำนวนจริงบวกใด ๆ

โดยอสมการ A.M.-H.M. สำหรับจำนวนจริงบวก a, b, c จะได้ว่า
$$(a+b+c)(\frac{1}{a}+\frac{1}{b}+\frac{1}{c})\ge 9$$ ดังนั้น $$a+b+c \ge \frac{9}{\frac{1}{a}+\frac{1}{b}+\frac{1}{c}} $$

แต่เนื่องจาก $\frac{1}{a}+\frac{1}{b}+\frac{1}{c} < 1$
แสดงว่า $a+b+c > 9$
นั่นคือ ถ้า a, b, c เป็นจำนวนเต็มบวกแล้วจะได้ว่า $a+b+c \ge 10$

กรณีที่ 1. $a+b+c=10$
10 = 2+3+5=2+4+4 =3+3+4
จะพบว่าไม่มี (a, b, c) ชุดใดที่ทำให้ $\frac{1}{a}+\frac{1}{b}+\frac{1}{c} > \frac{19}{20}$


กรณีที่ 2. $a+b+c=11$
11 = 2+3+6 = 2+4+5 = 3+3+5 = 3+4+4

จะพบว่าไม่มี (a, b, c) ชุดใดที่ทำให้ $\frac{1}{a}+\frac{1}{b}+\frac{1}{c} > \frac{19}{20}$

กรณีที่ 3. $a+b+c=12$
12=2+3+7=2+4+6=...
จะพบว่าถ้า (a, b, c) = (2, 3, 7) แล้ว $\frac{1}{a}+\frac{1}{b}+\frac{1}{c} = \frac{41}{42} > \frac{19}{20}$

นั่นคือ $a+b+c=12$ เป็นค่ำต่ำสุดที่เป็นไปได้ :cool:

Thgx0312555 19 ธันวาคม 2011 16:39

ควรจะมีเงื่อนไขเพิ่มเติมครับ ไม่งั้นจะไม่มีค่าต่ำสุด
เช่นลองแทนค่า a = 1.01

จะได้ $\frac{19}{20} < \frac{1}{a} < 1$

คราวนี้ลองแทนค่า b, c เป็นค่าน้อยมากๆ เช่น -1000000
จะได้
$\frac{1}{b}\approx \frac{1}{c}\approx 0$

แต่ยังน้อยกว่า 0 อยู่ครับ

นั่นคือยังตรงตามเงื่อนไข แต่ถ้าลดค่า b,c ลงเรื่อยๆ (ยังตรงเงื่อนไขอยู่) ก็จะได้ a+b+c น้อยลงตามไปด้วย

ดังนั้น a+b+c จึงไม่มีค่าต่ำสุดครับ

Real Matrik 19 ธันวาคม 2011 19:24

($a,b,c\in N$)
โดยไม่เสียนัยทั่วไปกำหนดให้ $a\geq b\geq c$ จะได้ว่า

$$\frac{19}{20}<\frac{1}{a}+\frac{1}{b}+\frac{1}{c}\leq\frac{1}{c}+\frac{1}{c}+\frac{1}{c}=\frac{3}{c}$$
$$c\leq\frac{60}{19}$$
นั่นคือ $c$ มีค่าที่เป็นได้อยู่ $3$ ค่าคือ $1,2,3$ (เห็นได้ชัดว่ากรณี $c=1$ ไม่สามารถเกิดขึ้นได้)

พิจารณากรณี $c=2$ จะได้ว่า
$$\frac{9}{20}<\frac{1}{a}+\frac{1}{b}\leq\frac{2}{b}$$
$$b<\frac{40}{9}$$
นั่นคือ $b$ มีค่าที่เป็นได้อยู่ $3$ ค่าคือ $2,3,4$ (เห็นได้ชัดว่ากรณี $b=2$ ไม่สามารถเกิดขึ้นได้)
ถ้า $b=4$ จะได้ว่า $4<a<5$ ซึ่งขัดแย้งกับที่ $a\in N$
ถ้า $b=3$ จะได้ว่า $a=7$ เป็นค่าที่น้อยที่สุดที่เกิดขึ้นได้
ดังนั้น กรณีนี้จะมี $(a,b,c)$ ที่สอดคล้องที่ทำให้ $\frac{1}{a}+\frac{1}{b}+\frac{1}{c}$ มีค่าน้อยที่สุดคือ $(2,3,7)$

พิจารณากรณี $c=3$ จะได้ว่า
$$\frac{37}{60}<\frac{1}{a}+\frac{1}{b}\leq\frac{2}{b}$$
$$b<\frac{40}{9}$$
นั่นคือ $b=3$ เท่านั้น ซึ่งส่งผลให้ $a=3$ เท่านั้น
แต่เราจะได้ว่า $\frac{1}{a}+\frac{1}{b}+\frac{1}{c}=1$ ซึ่งขัดแย้ง
ดังนั้นในกรณีนี้ไม่มี $(a,b,c)$ ที่สอดคล้อง

ดังนั้น ค่าของ $a+b+c$ ที่น้อยที่สถดที่ทำให้อสมการเป็นจริงคือ $12$

neo-freeman 19 ธันวาคม 2011 22:28

ขอบคุณทุกๆ แนวคิดครับ

ผมยังคงต้องค้นคว้าทำโจทย์อีกมากมายซินะ ^^


เวลาที่แสดงทั้งหมด เป็นเวลาที่ประเทศไทย (GMT +7) ขณะนี้เป็นเวลา 07:07

Powered by vBulletin® Copyright ©2000 - 2024, Jelsoft Enterprises Ltd.
Modified by Jetsada Karnpracha